You are on page 1of 36

LEGALEDGE TEST SERIES

Part of the most Comprehensive Classroom Training, Prep Content & Test Series across the Nation.
From the producers of A.I.R. 2, 3 and 5 in CLAT 2019.

MOCK ALL INDIA LAW ENTRANCE TEST 2020

MOCK AILET #03


Candidate Name : _________________
Duration : 90 Minutes
Batch : _________________
Max. Marks : 150
Contact No. : _________________
Centre Name : __________
Date of Exam : _________________

INSTRUCTIONS TO CANDIDATES

1. No clarification on the question paper can be sought. Answer the questions as they are.
2. There are 150 multiple choice objective type questions.
3. Each question carries ONE mark. Total marks are 150.
4. There is a negative marking of 0.25 marks for every incorrect answer.
5. Candidates have to indicate the correct answer by darkening one of the four responses provided, with
a BALL PEN (BLUE OR BLACK) in the OMR Answer Sheet.
Example: For the question, "Where is the Taj Mahal located?", the correct answer is (b).
The candidate has to darken the corresponding circle as indicated below :
(a) Kolkata (b) Agra (c) Bhopal (d) Delhi
Right Method Wrong Methods

6. Answering the questions by any method other than the method indicated above shall be considered
incorrect and no marks will be awarded for the same.
7. More than one response to a question shall be counted as wrong.
8. The candidate shall not write anything on the OMR Answer Sheet other than the details required and
in the spaces provided for.
9. After the Test is over, the candidate has to return the OMR Answer Sheet to the invigilator. The
T40_AILET03_ALL_01-2019-20

candidate should take the Test Paper along with them.


10. The use of any unfair means by any candidate shall result in the cancellation of his/her candidature.
11. Impersonation is an offence and the candidate, apart from disqualification, may have to face criminal
prosecution.
12. Electronic gadgets like mobile phones, pagers or calculators are strictly not permitted inside the Test
Centre/Hall.
13. The candidates shall not leave the hall before the Test is over.
MOCK AILET #03

SECTION-A : ENGLISH LANGUAGE

Directions (Q.1- Q.8) Read the following passage carefully and then answer the question that
follow.
"Art is thinking in images." Poetry is a special way of thinking; it is, precisely, a way of thinking in
images, a way which permits what is generally called "economy of mental effort," a way which
makes for "a sensation of the relative ease of the process." "Without imagery there is no art"—
"Art is thinking in images." These maxims have led to far-fetched interpretations of individual
works of art. Attempts have been made to evaluate even music, architecture, and lyric poetry as
imagistic thought.
Many still believe, then, that thinking in images is the chief characteristic of poetry. Consequently,
they should have expected the history of "imagistic art," as they call it, to consist of a history of
changes in imagery. But we find that images change little; from century to century, from nation to
nation, from poet to poet, they flow on without changing. Images belong to on one: they are “the
Lord’s”. The more you understand an age, the more convinced you become that the images a
given poet used and which you thought his own were taken almost unchanged from another poet.
The works of poets are classified or grouped according to the new techniques that poets discover
and share, and according to their arrangement and development of the resources of language;
poets are much more concerned with arranging images than with creating them. Images are given
to poets; the ability to remember them is far more important than the ability to create them.
Imagistic thought does not, in any case, include all the aspects of art nor even all the aspects of
verbal art. A change in imagery is not essential to the development of poetry. We know that
frequently an expression is thought to be poetic, to be created for aesthetic pleasure, although
actually it was created without such intent. Thus a work may be (1) intended as prosaic and
accepted as poetic, or (2) intended as poetic and accepted as prosaic. This suggests that the
artistry attributed to a given work results from the way we perceive it. By “works of art,” in the
narrow sense, we mean works created by special techniques designed to make the works as
obviously artistic as possible.

1. The primary purpose of the passage is to-


(a) Elucidate the difference between Art and Poetry.
(b) Challenge the basic aspects of art and poetry.
(c) Role of imagery in the development of poetry and use of Art as a technique.
(d) Understand the purpose of poetry and the basic structure.

2. Which of the following statement about image is supported by the information provided in the
passage?
(a) It’s important for a poet to create images.
(b) The earlier poets share their images with other poets.
(c) Creating imagery is pivotal to create an art.
(d) There is no change in the poetry as poets copy each other works.

3. Which of the following most accurately describes the AUTHOR opinion on Art?
(a) Art is a technique and the way we perceive differs from the way the writer writes.
(b) He believes that every art is created only for aesthetic pleasure.
(c) It’s integral to have all the aspects of art in an imagistic thought.
(d) He’s concern with the way people are trying to evaluate music, architecture and poetry as an
Art.

Head Office: 127, Zone II, MP Nagar, Bhopal |+91-9111555433| www.legaledge.in Page 2 of 36
MOCK AILET #03

4. Which of the following statement from the passage suggests that interpretation of work changes
with time?
(a) Attempts are being made to evaluate and interpret the different art forms.
(b) Work can be intended to be prosaic but accepted as poetic or the work maybe plagiarized.
(c) The change in images is little, from century to century, nation to nation.
(d) The artistry attributed to a given work results from the way we perceive it.

5. According to the passage, if you don’t understand an age and the art of that particular age, then
which of the following would be true?
(a) You’ll be convinced that there is hardly any change in the images used by the given poet and
the other poets.
(b) It would be difficult for an individual to interpret other forms of art.
(c) You might think that there the images used by the poet differ and the images were his own.
(d) You won’t be able to understand the special techniques used to make the work artistc.

6. According to the passage, which of the following statements is false?


(a) It’s important for the poet to remember the images rather than creating one.
(b) To develop a poetry, it’s crucial to create different images.
(c) Without imagery there is no art and to create an art it’s important to think in images.
(d) Poets share their techniques with other poets.

7. The author of the passage uses the expression “Art is thinking in images” (first para) primarily in
order to
(a) Define the importance of images while creating any art.
(b) Make the artist realize how important it is for them to create images.
(c) Suggest that images can be a good way to make your work famous and known.
(d) Lend support to the artist and tell the readers how important it is for them to remember the
images.

8. Which of the following statement about “work of art” can be inferred from the information in the
passage?
(a) If an artist use special techniques in his work, chances are that he’ll get famous.
(b) There are special techniques which are designed to make the work more artistic.
(c) The poets need to create images to have a better understanding about the artistic work.
(d) Techniques changes with time.

Directions (Q.9 - Q.11): In each of the following questions, a word is highlighted. Choose the
word which is a synonym of the highlighted word.

9. The swindler tried to evade the law.


(a) Circumvent (b) Churlish (c) Confront (d) Face

10. Early critics of Emily Dickinson’s poetry mistook for simple-mindness the surface of artfulness that
in fact she constructed with such craft.
(a) Astonishment (b) Cunning (c) Naïveté (d) Innocence

11. While others label her seemingly “carpe diem” attitude as carefree blithesomeness, I suspect it
mere fatuity.
(a) Imbecility (b) Frivolity (c) Cunning (d) Falsetto

Head Office: 127, Zone II, MP Nagar, Bhopal |+91-9111555433| www.legaledge.in Page 3 of 36
MOCK AILET #03

Directions (Q.12- Q13) Following are the questions based on the same words used as different
parts of speeches. Match the column numbered 1, 2, 3 and 4 with a, b, c and d. choose the best
combination.

12. Near
1. Adverb A. Her school is near the church
2. Verb B. A near relation must be considered
3. Preposition C. Come near and pay attention to what I say
4. Adjective D. The time nears

(a) 1(C), 2(A), 3(B), 4(D) (b) 1(B), 2(C), 3(A), 4(D)
(c) 1(C), 2(D), 3(A), 4(B) (d) 1(A), 2(D), 3(C), 4(B)

13. Wrong
1. Adjective A. Who is in the wrong?
2. Noun B. Wrong others; wrong yourself
3. Verb C You fixate on the wrong things
4. Adverb D. She heard wrong

(a) 1(B), 2(A), 3(D), 4(C) (b) 1(C), 2(A), 3(B), 4(D)
(c) 1(A), 2(B), 3(C), 4(D) (d) 1(D), 2(B), 3(C), 4(A)

Directions (Q.14 – Q.23): Each of the following question has a sentence, part or all of which is
underlined .Four sentences are given in the option. Choose the option which is correct
grammatically. Make sure the sentence is clear, exact and free from redundancy, ambiguity and
awkwardness.

14. For infrastructure to be created in such a fragile ecological zone, a rigorous analysis of possible
scenarios related to the impacts of climate change should be done which was not covered as a
part of the project, is proposed by the panel.
(a) Impacts of climate change should be done which was not covered as a part of the project, is
proposed by the panel.
(b) Climate changes and its impact which was not covered in the project, proposed by the panel.
(c) Changes in the climate change should be done that was not covered proposed the panel in
the project.
(d) Project which are proposed by the panel to look at the impacts of the climate change which
was not covered as a part of the project.

15. The Huen-Tsang account of the Gupta Era bears testimony to the fact that almost in every part
of Northern India, from the borders of Afghanistan to those of Burma, Fields have been regularly
cultivate and produce grains, fruits, and flowers in great abundance, but due the variation in the
distinct natural qualities, the crops also differ.
(a) Fields have been regularly cultivate and produce grains, fruits, and flowers in great abundance,
but due the variation in the distinct natural qualities, the crops also differ.
(b) Fields were regularly cultivated and produced grains, fruits, and flowers in great abundance,
but as the districts vary in their natural qualities they differ also in their crops
(c) The fields is been regularly cultivated and produced grains, fruits, and flowers in great
abundant but the district vary in their natural qualities they differ also in it’s crops.
(d) The variation in the crops differ because of the natural quality of the district and the fields
cultivated and produced grains, fruits, and flower as in abundance .

Head Office: 127, Zone II, MP Nagar, Bhopal |+91-9111555433| www.legaledge.in Page 4 of 36
MOCK AILET #03

16. Ultrasound applies to all sound waves which has the frequency above the audible range of human
being normal hearing that is around 20 kHz.
(a) Which has a frequency above the audible range of human being normal hearing that is around
20 kHz.
(b) That has a frequency above the range of audible human hearing that was around 20 kHz.
(c) With a frequency above the audible range of normal human hearing that is around 20 kHz.
(d) Around 20 kHz frequency, that is a audible range of a normal human being to hear.

17. Processing centre or AMPC is capable of sorting 10 lakh mails per day compared to the first class
and speed post mail of Delhi that has been handling more that 12 smaller mailing sorting centres.
(a) Which was earlier handled at more than 12 smaller mailing sorting centres.
(b) That has been handling more than 12 smaller mailing sorting centers.
(c) Where more than 12 smaller mailing sorting centers have been handle.
(d) In which 12 smaller mailing sorting centers was earlier handled

18. Automated Mail Processing Centre or AMPC is capable of sorting 10 lakh mails per day compared
to the first class and speed post mail of Delhi which was earlier handled at more than 12 smaller
mail sorting centres.
(a) An automated Mail Processing or AMPC was capable of sorting 10 mails per day compared
to the first class and speed post mail of Delhi
(b) Automated Mail Processing or AMPC is capable of sorting 10 mails per day compared to the
first class and speed post mail of Delhi
(c) Automated Mail Processing or AMPC is capable of sorting 10 mails per day compared by the
first class and speed post mail of Delhi
(d) Sorting 10 mails per day compared to the first class and speed post mail of Delhi Automated
Mail Processing or AMPC was capable and

19. I has a good time talking to Lisa. There was a lot of catching up to do. As for the movies, well, I
didn’t really care for them because none of them were interesting.
(a) As for the movies, well, I didn’t really care for them because none of them were interesting.
(b) I didn’t really care for it because none of the movies were interesting.
(c) As for the movies, well, I didn’t really care for them therefore none of them were interesting.
(d) As for the movies, well, I didn’t really care for them because none of them was interesting.

20. There are evidences of that the land revenue system in ancient India was based upon income
from land, thus rated according to the productivity and kind of soil.
(a) Their are evidences of that the land revenue system in ancient India was based upon
(b) There is evidences of that the land revenue system in ancient India was based upon
(c) There are evidences of that the land revenue system in ancient India was based upon
(d) Their are evidence of that the land revenue system in ancient India was based upon

21. I read the proof yesterday meticulously at home.


(a) I read the proof yesterday meticulously at home.
(b) I read the proof meticulously at home yesterday
(c) I read the proof at home meticulously yesterday
(d) I read yesterday the proof meticulously at home.

Head Office: 127, Zone II, MP Nagar, Bhopal |+91-9111555433| www.legaledge.in Page 5 of 36
MOCK AILET #03

22. It is a film that can be enjoyed by anyone, irrespective of age. I see it every year and yet I am not
jaded with it. The theatre have been jam-packed with persons of all age whenever that film is
shown.
(a) I see it every year and though I am not jaded with it. The theatre has been jam-packed with
persons of all ages whenever that film is shown.
(b) I see it every year and yet I am not jaded with it. The theatre have been jam-packed with
persons of all age whenever that film is shown.
(c) I see it every year and yet I am not jaded with it. The theatre has been jam-packed with persons
of all ages whenever that film is shown.
(d) I see it every year and yet I am not jaded with it. The theatre has been jam-packed with persons
of all age when that film is shown.

23. Rahul Mishra’s batting may be compared with the sales of a useful book: they score fast right from
the beginning.
(a) with the sales of a useful book: they score fast right from the beginning.
(b) to the sales of a useful book: they score fast right from the beginning.
(c) as the sales of a useful book: they score fast right from the beginning.
(d) on the sales of a useful book: they score fast right from the beginning.

Directions(Q.24 - Q.29) Following questions are based on various figures of speech.

24. Which of the following is not an example of epigram


(a) I'm not young enough to know everything
(b) I can resist everything but temptation
(c) Winners never quit and quitters never win
(d) I remain loyal to the crown

25. Which of the following is not an example of Litotes?


(a) I’m not forgetful that you served me well
(b) Art lies in concealing art.
(c) I’m not unfamiliar with poetry
(d) I’m a citizen of no mean city

26. Which of the following statement is an example of Metaphor?


(a) He eats like a pig
(b) He’s a walking encyclopedia
(c) The earth has swallowed all my hopes
(d) The owl hooted in the tree

27. Which of the following is an example of Metonymy?


(a) Suit for business executive
(b) The detective listened to her tales with a wooden face.
(c) Rishi sleeps like a baby all night.
(d) The car horn beeped loudly.

28. Which of the following is an example of Oxymoron?


(a) The comedian was seriously funny (b) The flowers begged for water
(c) My television hates me (d) He was playing to the gallery

Head Office: 127, Zone II, MP Nagar, Bhopal |+91-9111555433| www.legaledge.in Page 6 of 36
MOCK AILET #03

29. Which of the following statement is an example of Pun?


(a) One truth is clear: whatever is, is right.
(b) Every calendar’s days are numbered.
(c) The bells went ding.
(d) Rahul is as soft as teddy.

Directions (Q.30 - Q.35): Choose the best word to fill in the blanks.
Postmodernism (30)_____ the modernist ideals of rationality, virility, artistic genius, and
individualism, in favor of being anti- capitalist, contemptuous of traditional morality, and committed
(31)_____ radical egalitarianism. The most recent feature of Post modernism is the rise of Political
correctness and the attempt to (32)______dissenting opinion from the ranks of the academic/
artistic/ professional Brahmins caste, together with a systematic attack on excellence (33) ____all
fields. Post-modernism is an anti- Enlightenment position where (34)_______ believe that what
has gone before, as “Modernism”, is inappropriately (35)_________on Reason, Rationalism, and
Wisdom, and is furthermore, inherently elitist, non- multicultural and therefore oppressive.

30. (a) accepts (b) revive (c) rejects (d) approves

31. (a) for (b) to (c) on (d) among

32. (a) obtain (b) understand (c) purge (d) keep

33. (a) in (b) among (c) between (d) at

34. (a) antagonists (b) speakers (c) opponents (d) adherents

35. (a) dependent (b) independent (c) leant (d) standalone

Use for Rough Work

Head Office: 127, Zone II, MP Nagar, Bhopal |+91-9111555433| www.legaledge.in Page 7 of 36
MOCK AILET #03

SECTION - B : GENERAL KNOWLEDGE/CURRENT AFFAIRS

36. The gas that causes suffocation and death when coal or coke is burnt in a closed room is:
(a) Ethane (b) Methane
(c) Carbon dioxide (d) Carbon monoxide

37. The Transgender Persons (Protection of Rights) Bill 2019 has been passed by the Parliament.
Which of the following are features of the Bill?
(a) A transgender person will be given Right of Residence with parents and immediate family
members
(b) National Council for Transgender Persons (NCTP) will advise, monitor and evaluate measures
for the protection of their rights.
(c) Both (a) and (b)
(d) Neither (a) nor (b)

38. With which country is India signing the first maritime cooperation agreement between the two
countries?
(a) Switzerland (b) England (c) Sweden (d) USA

39. The Blue Flag Programme of the Indian Government is related to –


(a) River purification (b) Developing water-sports in India
(c) Developing beaches (d) Developing fisheries

40. English author Tony Joseph has won the 12th ‘Shakti Bhatt First Book Prize’. Which of his books
has won this prize?
(a) Early Indians: The Story of Our Ancestors and Where We Came
(b) Which of Us Are Aryans
(c) Who We Are and How We Got Here
(d) Modern South India: A Journey from the 17th Century to Modern Times

41. Who out of the following has won the Ballon d’Or Feminin Award for the year 2019?
(a) Alisson Becker (b) Megan Rapinoe
(c) Alex Morgan (d) Carli Lloyd

42. The Exercise Hand-in-Hand 2019 was held between India and ______ and was held in the state
of ________.
(a) Japan, Sikkim (b) China, Meghalaya
(c) Thailand, Manipur (d) Russia, Maharashtra
43. The song Jana-Gana-Mana composed by Rabindranath Tagore, was first published in January
1912 under the title of:
(a) Bharat Vidhata (b) Rashtra Jagrati
(c) Tatva Bodhini (d) Geetanjali.
44. Consider the following statements regarding Delhi Dialogue carefully:
1. The theme for this year’s Delhi Dialogue is "Advancing Partnership in Indo-Pacific”
2. This was the XIth edition of Delhi Dialogue
Which of the following statements regarding NGT is/are correct?
(a) Only 1 (b) Only 2
(c) Both 1 and 2 (d) Neither 1 nor 2

Head Office: 127, Zone II, MP Nagar, Bhopal |+91-9111555433| www.legaledge.in Page 8 of 36
MOCK AILET #03

45. Consider the following statements about a personality –


1. She/He is the longest-serving Indian American lawmaker
2. She/He has been named the chairman of a key US Congressional sub-committee that has
jurisdiction over India Identify the personality:-
(a) Kamala Harris (b) Ami Bera
(c) Raja Krishnamoorthi (d) Ro Khanna

46. Consider the women personalities of India –


1. Nirmala Sitharaman
2. Kiran Mazumdar-Shaw
3. Smriti Irani
4. Sonia Gandhi
5. Roshni Nadar Malhotra
Who out of the above have been named among the world’s 100 most powerful women by Forbes?
(a) Only 1, 2 and 3 (b) Only 2, 3 and 4
(c) Only 1, 2 and 5 (d) Only 2, 3 and 5

47. Consider the following statements about the Disha Bill –


1. This Bill was passed by the Union Parliament in light of the recent rape incident that happened
in Telengana
2. This Bill envisages the completion of investigation in seven days and trial in 14 working days,
where there is adequate conclusive evidence
Which of the above statements is/are incorrect?
(a) Only 1 (b) Only 2
(c) Both 1 and 2 (d) Neither 1 nor 2

48. Consider the following statements about the definition of kilogram -


1. The new prototype of kilogram is still not available in India which came into force in May 2019
2. According to the new definition of kilogram, it can now be measured in terms of the amount of
electric current
Which of the above statements is/are correct?
(a) Only 1 (b) Only 2
(c) Both 1 and 2 (d) Neither 1 nor 2

49. Which one of the following is considered as the most lasting contribution of the Rastrakutas?
(a) Kailasha, Temple
(b) Pampa, Ponna, Ranna, the three writer of Kannada Poetry and Kailasha Temple
(c) Patronage of Jainism
(d) Conquests

50. Consider the following statements about the India Skills Report 2019 –
1. The India Skills Report 2019-20 has been published by a joint effort of the NITI Aayog and the
MSDE
2. The report showed that 86.45 per cent students were found employable or ready to take up
jobs in 2019
Which of the above statement/s is/are correct?
(a) Only 1 (b) Only 2
(c) Both 1 and 2 (d) Neither 1 nor 2

Head Office: 127, Zone II, MP Nagar, Bhopal |+91-9111555433| www.legaledge.in Page 9 of 36
MOCK AILET #03

51. Study the following statements about the Climate Change Performance Index given in the options
and choose the correct one –
(a) India has joined the top ten countries in the rankings for the first time by securing the 8th rank
(b) Saudi Arabia has been ranked as the worst performing country
(c) The rankings are based on 14 indicators in four categories
(d) Sweden has secured the first rank in the index
52. The Industrial Development Bank of India (IDBI) was set up in:
(a) July, 1968 (b) July, 1966 (c) July, 1962 (d) July, 1964

53. Consider the following statements regarding the International Universal Health Coverage Day –
1. Every year 12 December is observed as the International Universal Health Coverage Day and
is promoted by the WHO
2. The theme for this year was ‘Keep the promise’
Which of the above statement/s is/are correct?
(a) Only 1 (b) Only 2 (c) Both 1 and 2 (d) Neither 1 nor 2
54. Consider the following statements regarding PSLV –
1. It marked its ‘Silver Jubilee’ launch by injecting India’s advanced radar imaging satellite RISAT-
2BR1 in its intended orbit.
2. The PSLV has launched many heavy-weight missions like the Mars Orbiter Mission,
Chandrayaan-1 and Chandrayaan-2 Missions etc.
Which of the above statement/s is/are incorrect?
(a) Only 1 (b) Only 2 (c) Both 1 and 2 (d) Neither 1 nor 2

55. Consider the following statements –


1. Nirav Modi has been declared as a Fugitive Economic Offender by a special court on a plea of
the Enforcement Directorate
2. He is the first businessman to be declared a fugitive economic offender
3. Nirav Modi is the key accused in the $2 billion Punjab National Bank (PNB) fraud case,
Which of the above statement/s is/are correct?
(a) Only 1 and 2 (b) Only 2 and 3 (c) Both 1 and 3 (d) 1, 2 and 3
56. The Citizenship (Amendment) Bill seeks to make people from Hindu, Sikh, Jain Buddhist, Christian
and Parsi faiths who enter India from Afghanistan, Bangladesh and Pakistan eligible for Indian
citizenship. Lots of protests have been seen against this proposed Bill. Which of the following are
the reasons for these protests?
1. It might nullify the 1985 Assam Accord
2. It links India’s citizenship to religion
Choose the correct answer out of the following –
(a) Only 1 (b) Only 2 (c) Both 1 and 2 (d) Neither 1 nor 2
57. Larry Page and Sergey Brin are stepping down as CEO and president, respectively, of Google
parent company Alphabet, making way for Google CEO Sundar Pichai to head Alphabet as well.
Which of the following statements is correct regarding the new role of Sundar Pichai?
(a) Sundar Pichai will now be the President of Alphabet and Google
(b) Sundar Pichai will be the President of Alphabet and CEO of Google. The post of Alphabet’s
CEO will remain vacant
(c) Sundar Pichai will be the CEO of Alphabet and Google. The post of Alphabet’s President will
remain vacant
(d) Sundar Pichai will now be the President and CEO of Alphabet and Google both

Head Office: 127, Zone II, MP Nagar, Bhopal |+91-9111555433| www.legaledge.in Page 10 of 36
MOCK AILET #03

58. The Supreme Court prima facie agreed to the Airports Authority of India’s (AAI) request to build
an extra terminal at Agra airport, but forbid any increase in air traffic. What was the reason for this
limitation?
(a) There is not enough infrastructure to add more air traffic
(b) It would have detrimental effect to Taj Mahal
(c) Both a and b
(d) Neither a nor b
59. The Gujarat government decided to make wearing helmets optional within city limits. What was
the reason given for this change?
(a) The cities of Gujarat has very good roads, which reduces the chances of accident
(b) People don’t cover long distances on two-wheeler
(c) There are too many cases of violation of rules
(d) The fines for violation of the helmet-rule is too hefty
60. The furore surrounding fee hikes at the Jawaharlal Nehru University has spurred deeper questions
about the quality of university education. India’s higher education system is structurally flawed and
underfunded. This crisis puts at risk India’s –
1. Innovation and human capital
2. Labour Productivity
3. GDP growth
Choose the correct code out of the following –
(a) Only 1 and 2 (b) Only 2 and 3 (c) Only 1 and 3 (d) 1, 2 and 3
61. The Personal Data Protection Bill has been approved by the Parliament. It seeks to bring in a
provision for the right to be forgotten. What does this mean?
(a) The right to restrict or prevent continuing disclosure of personal data by the citizens
(b) The right to have negative private information about a person to be removed from internet
searches
(c) The right to citizens to categorise information as critical personal data, which will be shielded
from scrutiny
(d) The right to citizens to be left undetected by the government agencies
62. The Kachin Independence Army (K.I.A.) is the military arm of the Kachin Independence
Organization (K.I.O.), a political group based in?
(a) China (b) Myanmar (c) India (d) Thailand
63. Consider the following statements –
1. The Centre has asked the Supreme Court to refer to a seven-judge Bench whether the creamy
layer concept should apply to the SCs/STs while providing them reservation in promotions
2. A 2006 judgement of the Supreme Court had upheld the application of the creamy layer
principle in promotions.
Which of the following cases is related to it?
(a) M. Nagaraj Case (b) Shreya Singhal Case
(c) Waman Rao Case (d) S. R. Bommai Case
64. Drugs Controller General of India (DCGI) has asked all states and Union Territories to prohibit
sale of drugs by online pharmacies as per the Delhi High Court order. Why has this been done?
(a) e-Pharmacies hoard medicines, which increase the prices
(b) e-Pharmacies are leading to deterioration of the pharmaceutical shops
(c) e-Pharmacies charge exorbitant rates for emergency drugs
(d) e-Pharmacies are operating in the country without a drug licence

Head Office: 127, Zone II, MP Nagar, Bhopal |+91-9111555433| www.legaledge.in Page 11 of 36
MOCK AILET #03

65. The first human heart transplant in the history was done in 1967 in which among the following
countries?
(a) South Africa (b) Pakistan (c) USA (d) Irelan

66. There have been reports that China is trying to ‘homogenize’ the Uighurs. What does ‘homogenize’
mean in this context?
1. Give them proper training to make them at par with the mainland Chinese
2. ‘De-radicalise’ the Uighurs
3. Force the Uighurs to give up their identity
Use the following codes to give the correct answer.
(a) Only 1 and 2 (b) Only 2 and 3 (c) Only 1 and 3 (d) 1, 2 and 3

67. Consider the following statements about a ranking-report that has been published –
1. The list has been topped by Norway out of 189 countries
2. India has been ranked 129 out of the 189 countries
Which of the following organisations has published the report described above?
(a) World Economic Forum (WEF)
(b) United Nations Framework Convention on Climate Change (UNFCCC)
(c) United Nations Development Programme (UNDP)
(d) International Monetary Fund (IMF)

68. Soil-less agriculture refers to:


(a) Sericulture (b) Hygroponics (c) Hydroponics (d) Inter-cropping

69. Consider the following statements about an organization –


1. The organisation’s dispute settlement mechanism is on the brink of collapsing
2. The main reason for the above stated situation is that the US has blocked the appointments of
new members
Who out of the following is the head of the above-described organization?
(a) Kristalina Georgieva (b) Roberto Azevedo
(c) José Ángel Gurría (d) David Malpass

70. Consider the following statements about a programme launched by the Union Government –
1. It has been launched by the Ministry of Environment, Forest & Climate Change
2. It aims to sensitise the people and students, in particular, about climate change and global
warming
Which of the following is the name of the programme?
(a) Green Social Responsibility (b) Green Good Deeds
(c) Go Green (d) Green Champions

Use for Rough Work

Head Office: 127, Zone II, MP Nagar, Bhopal |+91-9111555433| www.legaledge.in Page 12 of 36
MOCK AILET #03

SECTION – C: LEGAL APTITUDE/REASONING

71. Legal Principles:


I. An offer is an expression of willingness to contract on specified terms, made with the intention
that it is to become binding as soon as it is accepted by the person to whom it is addressed.
II. The communication of an offer is complete when it comes to the knowledge of the person to
whom it is made.
III. The communication of an acceptance is complete, as against the proposer, when it is put in
a course of transmission to him, so as to be out of the power of the acceptor; as against the
acceptor, when it comes to the knowledge of the proposer.
IV. A proposal may be revoked at any time before the communication of its acceptance is
complete as against the proposer, but not afterwards.
V. An acceptance may be revoked at any time before the communication of the acceptance is
complete as against the acceptor, but not afterwards.
VI. The acceptance should be made in the mode of the communication prescribed by the
proposer. Any departure from the mode of communication fill result in no communication of
acceptance.
Factual situation: Rajeev has lived most of his life in the city and had made many friends. But
with increasing rush in the city, he realized that the city life is not made for him. He has earned
enough to sustain himself and his family. Therefore, he decided to move to the village and take
up a smaller job. He owned a huge bungalow in the city and was worried about it. He remembered
that his friend Shameek had always been interested in building a similar house as Rajeev’s. On
1st February, Rajeev sent a letter to Shameek which said “Dear Shameek, I have decided to sell
my bungalow in the city and move to village. I thought you might consider buying it from me. I will
sell it to you for its current market value which is Rs. 3 crore. Call me by 10th February. I will keep
this proposal open, and will not withdraw it, until after that date.” Next day, Shameek was
discussing about his interest in buying the bungalow with his sister Mehak. Mehak responded that
a friend of hers, Kamini has also received the offer from Rajeev to purchase the bungalow.
Shameek on realizing that he may lose the offer, immediately went to his room and wrote the letter
of acceptance addressed to Rajeev. He deposited the letter in the mailbox at 10:30 A.M. At 11
A.M., Rajeev entered into the written agreement with Kamini for the purchase of the bungalow for
Rs. 3 crore. At 2:00 P.M. on 2nd February, Shameek called Rajeev requesting survey of the
bungalow. Rajeev then informed Shameek that he had already sold the bungalow to Kamini. Upon
hearing this, Sameer exclaimed that “We have a deal. I sent you the acceptance via mail this
morning.”
Which of the following is true according to the facts?
(a) There is no contract between Shameek and Rajeev because the letter by Rajeev to Shameek
is merely an invitation to offer and the letter by Rajeev to Shameek is the offer.
(b) There is clear contract between Rajeev and Shameek because Rajeev’s letter was in clear
terms an offer for sale which had been accepted by Shameek through letter posted by mail at
10:30 before Rajeev had entered into agreement with Kamini.
(c) There is no contract between Rajeev and Shameek because Rajeev clearly stipulated “Call”
as a medium to communicate acceptance of the offer which was not followed by Shameek
while accepting the offer.
(d) There is a contract between Rajeev and Shameek because acceptance of Shameek’s offer
was done before Kamini’s acceptance as per the postal rule which is that acceptance is
complete against the offeror as soon as it reaches out of control of the offeree.

Head Office: 127, Zone II, MP Nagar, Bhopal |+91-9111555433| www.legaledge.in Page 13 of 36
MOCK AILET #03

72. Legal Principles:


I. Consideration is an act, forbearance, or return promise bargained for and given in exchange
for the promise.
II. A promise is not enforceable unless it is supported by consideration.
III. Past consideration is not a valid a consideration and has no legal value.
IV. Consideration must have some value in the eyes of law.
V. Consideration must be sufficient and not adequate.
Factual situation: John was a big businessman. He loved his son Hank and hoped that Hank
would someday join his father’s business. Hank was carefree and didn’t care much about his
father’s business. He had a habit of spending his fathers’ money on luxuries and in hanging out
with his friends. Each Sunday, Hank and his friends used to race on their bikes on the highway.
One Sunday, Hank went to race with his friend but got into an accident resulting into severe injuries
to Hank. Elvis, who is a licensed physician was passing on that highway and happened to be there
at the time of accident. He immediately went out of his car to check on Hank and found Hank
unconscious. Elvis immediately treated Hank at the scene and then took him to his hospital. A few
days later, John found about Hank’s accident and his whereabouts and came to the hospital. He
was very thankful to Elvis and told him that “If you continue treating my son, I will pay all of your
fees i.e. both for the services already rendered and for the future services that you will deliver to
Hank.” Elvis thanked John and told him to have patience while he continues treating Hank. The
treatment continued but Hank didn’t regain consciousness. Three days later, he died. John was
shocked at his son’s death. Elvis gave John the bill of treatment which amounted to $5000. The
value for Elvis’ services before his conversation with John was $ 3000 and after the conversation
was $ 2000. John was angry and refused to pay for any of the services. Which of the following is
true?
(a) Elvis is entitled only upto $ 2000 because services rendered by Elvis before John made a
promise to him were past services which is not a valid consideration.
(b) Elvis is entitled to payment of $ 5000 for the services rendered by him both prior and after the
promise because John’s promise to pay for future services acts as consideration for both past
and future services.
(c) Elvis is not entitled to any of the payment because he failed in performing his part of the
contract resulting into death of Hank.
(d) None of the above.

73. Legal Principles:


I. Where a seller accepts part payment of the debt owed to them under a contract, there is
nothing to prevent the seller claiming the balance due to him at a later date.
II. The agreement to accept part payment as a full satisfaction of the contract would be binding if
the buyer, at the seller’s request, had provided some fresh consideration.
III. The doctrine of promissory estoppel operates where one party has made a representation that
they do not intend to enforce their strict legal rights, made with the intention that the other party
will rely on the representation, and that party does indeed rely on the representation without
providing consideration.
Factual situation: Katie is an accountant for the Missandei Company. She is a hard working
person and has been working in the company for several years. She had always dreamt of a
buying a car but because of lack of funds, couldn’t buy one. Therefore, when Katie was promoted,
she planned to buy a second hand car from her colleague Riz. Riz sold the car to Katie for £3,000
and agreed for payment in two instalments. She had paid an amount of £1000 at the time of buying
and agreed to pay £500 after six months i.e. on 1st August and rest of £1,500 after a year. She
paid her first instalment on time which was due on 1st August. But in December, Katie lost her job
at Missandei Company. Her second instalment was due on 1st February, which she failed to pay

Head Office: 127, Zone II, MP Nagar, Bhopal |+91-9111555433| www.legaledge.in Page 14 of 36
MOCK AILET #03

because of the loss of job. Katie never disputed that she owed him the money but was not in the
condition to pay the second instalment. Riz was also a good friend of Katie and felt sorry for her
as she had lost her job not long after buying his car. Therefore, he told her that if Katie could pay
the half of what she owed to Riz i.e. £750 by the following Saturday, he would let her off with the
rest. Katie was overwhelmed by Riz’s offer. She could not find a way to thank him. She knew that
Riz loves dogs. Therefore, she paid Riz £750 on Thursday and also gifted him a dog “in
consideration of his kindness”. Riz kindly accepted both the payment and the dog. After a week,
Katie found that she had been successful in securing job at Thompson Company. She received
the offer letter from Thomson which stipulated the terms of her services and her salary. She was
offered a much better salary than her old job on the first month of her job itself. She immediately
accepted the offer and started working at Thompson Company. Riz heard about her new job and
realized that she would be earning much more than he is. Therefore, he wished to claim the rest
of the payment for the car which Katie had agreed to pay initially while buying the car. Katie refused
the payment of the rest of the amount.
(a) Katie is not bound to pay because doctrine of promissory estoppel would prevent Riz from
enforcing his right to claim payment since Katie relied on his representation and paid £750 as
a satisfaction for second instalment.
(b) Katie has to pay because acceptance of part payment of the debt owed to a person under a
contract cannot prevent the person from claiming the balance at a later date.
(c) Katie is not bound to pay because she had provided fresh consideration for acceptance of part
payment as satisfaction for full payment in the form of a dog.
(d) None of the above.

74. Legal Principles:


I. Every person is competent to contract who is of the age of majority according to the law to
which he is subject, and who is of sound mind and is not disqualified from contracting by any
law to which he is subject.
II. An agreement entered into with a minor is void ab initio. Any agreement entered into with a
major is valid.
III. A person who supplies necessities of life to a minor is entitled to be reimbursed for such supply.
Factual situation: Pallavi is a hard working student. She has just passed her 9th standard. She
secured 2nd rank in the class. She was crying at home for failing to secure 1st rank. She told her
father that she needs study table to secure 1st position in the class. She complained that because
she had to study on her bed, she would fall asleep early and therefore got 2nd rank. The study
table would help her remain focussed on studying for longer hours. Pallavi insisted to her father
to buy a study table so that she can secure best marks in her board examination of class 10th.
Pallavi’s father understood the importance of the table but refused to buy one because of his
financial condition. He told Pallavi to try focussing on her studies without a study table and
explained that securing 2nd position is also not bad. Pallavi realized that the study table is
necessary for her to top the class and therefore visited Furnster furniture shop. Pallavi knew that
if she buys the table, her father would definitely pay for it. The shop was owned by Mr. Bachchan.
Pallavi looked around the shop and found the cheapest study table for 6000/- Rupees. When Mr.
Bachchan saw Pallavi, he realized that Pallavi hasen’t attained the age of majority. Thus, when
Pallavi offered to buy the study table for 6000 Rupees, Mr. Bachchan inquired of about who would
pay for the table. She told him that she would ensure that her father will pay for the table. Relying
on Pallavi’s promise Mr. Bachchan sold the table to Pallavi and sent the bill for the same to her
father. Pallavi’s father Anil refused to pay for the study table. Can Mr Bachchan claim payment for
the study table bought by Pallavi?

Head Office: 127, Zone II, MP Nagar, Bhopal |+91-9111555433| www.legaledge.in Page 15 of 36
MOCK AILET #03

(a) Mr. Bachchan is entitled to the payment of Rs. 6000 for the study table because the table was
a necessity for Pallavi’s education.
(b) Mr. Bachchan is not entitled for the payment of Rs. 6000 for the study table because the table
was not a necessity for Pallavi’s education.
(c) Mr. Bachchan is entitled to the payment for the study table because he sold the table relying
on the promise that Anil would pay for the table.
(d) Mr. Bachchan is not entitled for the payment because agreement with Pallavi is the agreement
with a minor which is void ab initio.

75. Legal Principles:


I. There cannot be a stranger to a contract.
II. The doctrine of privity of contract is a common law principle which provides that a contract
cannot confer rights or impose obligations upon any person who is not a party to the contract.
The premise is that only parties to contracts should be able to sue to enforce their rights or
claim damages as such.
III. Doctrine of privity of contract provides that a third person is not entitled to demand performance
of the contract.
IV. If a contract itself is made for the benefit of a third person, then such third person can sue for
the performance of the contract.
V. Consideration for an agreement can proceed from third party.
Factual situation: Mr. Bart Bass was a big and successful businessman in New York. He had
business and investment across the world. He had invested in many luxury hotels and restaurants.
He had a son named Bucky who died few years back. Buckey had a son named Chuck. After
Bucky dies, Chuck was in the care of his grandfather Bart Bass. When Chuck was 15, Bart Bass
got a heart attack and recovered from it. He had almost seen death. Since then, he strated
worrying about Chuck because he was too young. Therefore, Bart Bass created a trust for Chuck
and made his most trusted friend, Eleanor Waldorf as the trustee. Eleanor Waldorf was the
Manager of Bart Bass’ business in New York. The trust was named as Buck Bass Trust. Mr. Bart
Bass and Eleanor Waldorf had entered into an agreement where it was specified that all the
income from Bart Bass’ business in France and New Jersey would go to the trust fund. Moreover,
it said that the 60% of income from the business in New York would go to the trust fund.
Additionally, the trust fund would be used to pay for Chuck Bass’ education and necessities. After
Bart Bass died, Eleanor took control of his business. She started keeping 70% of New York’s
business with herself and only transferred 30% to the trust fund. Chuck on attaining the age of
majority took control of his grandfather’s business. He found out about Eleanor’s conduct and
sued her to reimburse the money to the trust fund.
(a) Chuck cannot sue Eleanor because he is a stranger to the contract between Mr. Bart Bass and
Eleanor Waldorf.
(b) Chuck can sue Eleanor for damages because he is the intended beneficiary of the contract
between Bart Bass and Eleanor Waldorf.
(c) Chuck cannot sue Eleanor because the trust created was to save money for further investment
and Chuck is not privy to the same.
(d) None of the above.

76. Legal Principles:


I. Consensus ad idem is the foundation of contracts to become enforceable.
II. It means that for a contract to become enforceable, meeting of minds is required.
III. Two or more person are said to consent when they agree upon the same thing in the same
sense.

Head Office: 127, Zone II, MP Nagar, Bhopal |+91-9111555433| www.legaledge.in Page 16 of 36
MOCK AILET #03

Factual situation: Serena wanted to buy a doll house for her daughter’s 4th birthday. She visited
to various shops to buy a doll house but could not find the one she needed. Serena’s friend Blair
was an expert in the doll houses. She herself makes them and sell it to people. Serena approached
Blair and told her how she has been looking for a perfect doll house for her daughter but couldn’t
find one. Serena said to Blair that “I want “The Doll House” for my daughter which would make
her happy.” Serena emphasized to tell Blair that she wants a perfect one of a kind doll house. Blair
replied that “I understood and you can come see it tomorrow near Lindsay Palace and then buy
it.” Serena got really excited and agreed to take it and said that she has already bought it and paid
$1000 at that moment. Blair had a house which she inherited from her mother. Blair had named it
“The Doll House”. It was near Lindsay Palace. She had put it on sale and advertised about it in
newspaper. She invited Serena to the house and showed her around the house. Serena really
liked the house. Blair asked for the rest of the money.
(a) The contract is not enforceable because Serena agreed for a toy doll house and Blair agreed
for her house named “The Doll House”.
(b) The contract is enforceable because Serena wanted to buy Blair’s inherited “The Doll House”
as she had read about it in newspaper.
(c) The contract is not enforceable Serena did not intend to create legal relations when she agreed
to see the Doll House.
(d) The contract is enforceable because Serena had asked for “The Doll House” and it was her
fault in not clarifying about it.

77. Legal Principle:


I. Intention to deceive must be present to establish the tort of deceit.
II. A Corporation is liable for the tortuous act of its employee, if the act leading to the liability was
done within the course of employment.
III. An act is in the course of employment even if it is an unauthorized or wrongful mode of doing
an authorized act.
Factual Situation: In the city of Jorah, people had immense faith in Buddha. Madagascar was a
company producing Buddha models. It had a manufacturing unit and a selling unit. Madagascar
produced both kinds of Buddha model. One of original metal and other having metal like look. The
Buddha model of original metal was costlier than the non-metal Buddha. Simba was the newly
appointed CEO of the company and started taking all decisions regarding manufacturing and
selling of the Buddha. He declared to the employees that the company will no longer be producing
the non-metallic Buddha models. Madagascar also had online platform in the city, whereby it had
home delivery facility. The facility provided the option to customers to check the product on
delivery and then take it. Tarzan was one of the delivery boy. Mr. Winnie once ordered a metallic
Buddha online. Tarzan delivered the Buddha. Mr. Winnie enquired whether the Buddha model is
made of pure metal. Tarzan assured that it is pure metal and that the company no longer produces
non-metallic models. Later, when the Buddha model fell off the table and broke, Mr. Winnie
realized that the model was not of pure metal. He sued Tarzan and the Madgascar Company for
tort of deceit. Which one the following is true?

(a) Tarzan is liable because Tarzan lied about the product. Madagascar is liable because delivery
of Buddha was in the course of employment.
(b) Tarzan is not liable because he was just a delivery boy who did not have knowledge about the
product but Simba is liable for giving out false information.
(c) Tarzan is liable because he had intended to sell the product at any cost. Madagascar is liable
because Tarzan was in the course of employment while delivering the Buddha.
(d) Tarzan is not liable because he relied on information by the company that it won't produce non-
metallic buddhas. Therefore, Madagascar is also not liable for tort of deceit.

Head Office: 127, Zone II, MP Nagar, Bhopal |+91-9111555433| www.legaledge.in Page 17 of 36
MOCK AILET #03

78. Legal Principles:


I. False imprisonment occurs when a person intentionally restricts another person’s movement
within any area without legal authority, justification or consent.
II. The necessary intent for the purpose of false imprisonment is the intent to confine.
III. False imprisonment is a form of trespass to person.
IV. Lawful detention by the police is not false imprisonment.
Factual situation: Manika went to shop at H&M store. She was wearing a gold necklace when
she entered the shop. While looking at a bell bottom pants, she felt somebody remove her gold
chain from her neck. She instantly alarmed the staff of H&M store and they closed all the exit
points and stopped everyone at their position. The staff of H&M proceeded to check the camera
recording but due to some technical glitch they couldn’t access the videos. On request of Manika,
they conducted search of all those people who were readily willing to get them searched. For the
people, who did not allow the search, security staff was called. On searching Vidushi’s bag, who
volunteered for search, they found the Manika’s necklace. They asked Vidushi about it and she
refused to acknowledge taking of the necklace. She said she had no idea about how it came to
her. Vidushi was asked to wait in office until they conduct their inquiry and she went to the office
and waited. Vidushi thus sat in the office for around an hour. By then the video could be accessed
and the staff found that somebody else had slipped the necklace in Vidushi’s bag and that Vidushi
was innocent. Vidushi felt violated for being made to sit in the office for an hour and therefore filed
a suit for false imprisonment.
(a) Staff of H&M are liable for false imprisonment because they confined her in the office without
any legal authority as Vidushi had not committed theft.
(b) Staff of H&M are liable for false imprisonment because they were not Police who has lawful
authority to imprison a person.
(c) Staff of H&M are not liable for false imprisonment because Vidushi consented to sit office
without any complain.
(d) Staff of H&M are not liable for false imprisonment because she was not confined in a jail or
prison but was merely asked to wait in the office by the staff.

79. Legal Principles:


I. A person is liable for the tort of negligence if he breaches a legal duty of care he owes to the
plaintiff, and the plaintiff suffered a damage resulting from this breach.
II. One can claim damages from another if one breaches duty of care owed to another.
III. If someone willingly places themselves in a position where harm might result, knowing that
some degree of harm might result, they will not be able to bring a claim against the other party
in Tort.
IV. The harm resulting from the breach of duty must be reasonable and foreseeable by the
defendant.
Factual situation: In a shopping complex, all items were displayed on the racks to buy. There
were different rows for different kinds of items. Raju had diabetes and kind of extra sugar was
extremely harmful for his health. He went to the shopping mart and was looking for cranberry
juices with no added sugar. He asked the Prateek, who was a shopping mart staff about the
cranberry juices and Prateek took him to the row where all the juices were kept. Prateek helped
him find “Teal” brand of cranberry juice. Raju bought the juice and drank it in the evening. The
bottle of cranberry juice said that the juice was best before 12 months from the date of
manufacturing. Twelve months had already passed when Raju bought the bottle of juice. At night
after drinking the juice, Raju felt extremely sick and was urgently taken to the hospital. The doctor
found that his sickness was due to the added sugar content in the cranberry juice. Raju when
discovered that the juice was expired sued the shopping mart for negligence.

Head Office: 127, Zone II, MP Nagar, Bhopal |+91-9111555433| www.legaledge.in Page 18 of 36
MOCK AILET #03

(a) Shopping mart is liable for the harm caused to Raju because they sold an expired product.
(b) Shopping mart is liable for the harm because they sold a product with added sugar to Raju
who was diabetic.
(c) Shopping mart is not liable because they could not reasonably foresee that Raju was diabetic
and that the Teal juice would be harmful for him.
(d) Shopping mart is not liable because Raju voluntarily bought the juice and had put himself in
the position of harm knowingly.

80. Legal Principles:


I. Unreasonable interference with person’s use or enjoyment of his/her land amount to private
nuisance.
II. The plaintiff must have required interest in the land in order to have the capacity to sue.
Factual situation: Mr. Cooper and Mrs. Cooper were newly married. They had planned to live in
outskirts of the town away from the rush of the city. Therefore, Mrs. Cooper bought a house in
outskirts of North Wales. Their house was in the middle of a huge agricultural area. People in that
area were mostly engaged in animal farming. The life in that area was calm and people were free
of the stress of the city life. Both Mr. and Mrs. Cooper moved to the new house and were living
happily. They had also let in their friend Leah for short period of time. The land on the east side of
Mrs. Cooper’s house was being used for pig farming for the pork industry. Additionally,
construction of a house which was on tight schedule was being done on the land on the west side
of Mrs. Cooper’s house. Mrs. Cooper and Leah had a habit of taking sleeping pills and therefore
were unaffected by the construction going on at night. But Mr. Cooper could not sleep all night
because of the noise from the construction. Leah had a room on the second floor with huge
window. Whenever the wind blew, the smell from the pig farm made it unbearable for her to stay
in her room. She had to close her window making her unable to access fresh air. Thus, she was
very irritated and consulted the farm owner. The farm owner could not do anything to help her
because he had been doing pig farming for many years on that land. Thus, Mr. Cooper and Leah
filed suit for injunction on the ground of tort of private nuisance.
(a) Mr. Cooper’s and Leah’s claim would not survive because he does not have proprietary interest
in the house. Only Mrs. Cooper can bring the claim for tort of private nuisance.
(b) Mr. Cooper’s claim would survive because he being the spouse of Mrs. Cooper has beneficial
interest in the house and has construction noise at night is unreasonable interference.
(c) Leah’s claim would survive because the smell from the pig farm amount to unreasonable
interference with her enjoyment of the house.
(d) Both (b) and (c).

81. Legal Principles:


I. Borrowed servant rule is a common law legal doctrine that the employer of a borrowed
employee, rather than the employee’s regular employer, is liable for the employee’s actions
that occur while the employee is under the control of the temporary employer.
II. The temporary employer, called the special employer, is responsible for directing the work of
the borrowed worker, and the borrowed worker provides services for the special employer
rather than his or her regular employer.
III. When both the regular employer as well as special employer are in control of the employee,
both of them would be liable for employees’ actions.
Factual situation: Pragya owned a big house with a big lawn in it. Rajat Baba is a gardener who
took care of Pragya’s lawn. He made a beautiful garden in Pragya’s house with all kinds of
beautiful flowers. Pragya had advised him about the kind of flowers to be planted and how to plant
them. Pragya threw a party at her house and people appreciated her lawn. They asked about the
gardener and Pragya told them about Rajat Baba. Ankita, a close friend of Pragya asked Pragya

Head Office: 127, Zone II, MP Nagar, Bhopal |+91-9111555433| www.legaledge.in Page 19 of 36
MOCK AILET #03

to borrow her gardener for few days so that he can make a beautiful garden at Ankita’s house.
Pragya explained Rajat Baba of what kind of plants to sow and how to plant the flowers. Rajat
Baba while planting the trees, injured a passing by kid with the spading fork. The kid’s mom filed
suit against Ankita and Pragya.
(a) Only Ankita is liable because she had borrowed Rajat Baba and he was providing services to
Ankita.
(b) Only Pragya is liable because she was in control of the actions of Rajat Baba.
(c) Both Pragya and Ankita are liable because both of them were in control of Rajat Baba’s actions.
(d) Ankita is not liable because she did not have any control over Rajat Baba’s actions. He was
merely working on her lawn.

82. Legal Principles:


I. In the law of torts, if any person commits any wrongful act which causes injury to another
person, he is held liable and has to pay damages or provide some other remedy which the
Court determines, to the victim of such an act.
II. A person is liable for the tort of negligence if he breaches a legal duty of care he owes to the
plaintiff, and the plaintiff suffered a damage resulting from this breach.
III. In case a person gives his consent to doing of an act which leads to him getting injured, then
even if an injury is caused by the other person, he cannot claim any damages from that person
because the act was one for which he voluntarily consented. This is the principle of volenti non
fit injuria.
Factual situation: Mouse had two children. Mouse’s first kid is an eight year old boy named
Giraffe and he has a mental disorder. It is because of his mental disorder that Mouse does not
take him out and he lives under his grandmother’s supervision. Mouse’s second child is a one
year old daughter named Piggy and she gets all the attention from Mouse. One day Mouse and
the grandmother took both Giraffe and Piggy to a park nearby. Mouse met Shepherd, another
parent and started talking to him. Shepherd was good looking and attractive. Mouse left Piggy to
play on the grass believing that grandmother would look after Piggy. Mouse got swayed away
while talking to Shepherd and got focussed on the conversation with him. Shepherd suddenly
realized that Piggy had gone away and has reached the road nearby. He ran towards the road to
save Piggy. Mickey was driving his van on the road safely and within the speed limit. Shepherd
managed to save Piggy but because of too much forward momentum collided with Mickey’s van.
His leg was injured. Shepherd filed suit against Mouse and Mickey.
(a) Mouse has to pay damages to Shepherd for her negligence because she owed duty of care to
exercise reasonable supervision over the child as it may have caused injury to people nearby.
(b) Mouse does not have to pay damages to Shepherd because he voluntarily assumed the risk
and consented to the injury while saving Piggy.
(c) Bob has to pay damages because he was negligent while driving his van and thus causing
injury to Shepherd.
(d) Bob is not liable to pay damages because Shepherd voluntarily came in front of his van.

83. Legal Principles:


I. Copyright is a natural right. The work need not be registered in order to be copyright protected.
II. There is Copyright over the expression and not the idea.
Factual situation: Mekta Kapoor planned to introduce a reality TV show on Swayamvar to the
television industry. She wrote the whole concept of the reality TV show on Swayamvar where she
provided that the bride to be will chose her husband based on participating men being able to
complete certain tasks. The task would involve their ability to handle a panic situation, their ability
to do household chores and their attitude. She showed her idea to the producer Methalani and
requested to bring the show to the TV. Methalani told her that he would think about the show.

Head Office: 127, Zone II, MP Nagar, Bhopal |+91-9111555433| www.legaledge.in Page 20 of 36
MOCK AILET #03

Later, he himself released a reality TV show on the concept of Swayamvar where the bride to be
will choose her husband based on his physical strength. Mekta Kapoor on seeing the show found
that Methalani had copied her concept of reality TV show on Swayamvar. Mekta Kapoor sued
Methalani for copyright infringement.
(a) Methalani is liable because Mekta Kapoor has written down her idea forming an expression
and Methalani has copied such expression.
(b) Methalani is not liable because he merely copied the idea of reality TV show on Swayamvar.
The idea of swayamvar had been long existing in the myths.
(c) Methalani is liable because he dishonestly did not give credit to Mekta Kapoor and released
the show as his own.
(d) Methalani is not liable because he had different idea from what Mekta Kapoor had on
Swayamvar.

84. Legal Principles:


I. An act which causes death and is done with the intention of causing death, or with the intention
of causing such injury as is reasonably likely to lead to death, amounts to murder.
II. If an act that otherwise amounts to murder is done under grave and sudden provocation, it only
amounts to culpable homicide not amounting to murder.
Factual situation: Pratyush was taking walk with his father. While they were taking walk, he saw
two men passing by shoot his father with gun. His father died. He immediately took out a knife
from his pocket and tried to stab one of the shooters. But he failed and the shooters ran away. He
recognized the shooters. He reported the incident at the police station and went to his friends.
They all plotted to kill the murderer of his father by attacking the murderer at his own house.
Partyush and his friends went to the shooters house and broke in the house. Pratyush first killed
the murderer’s wife who was trying to protect her husband and then Pratyush killed the murderer.
Is Pratyush liable for murder?
(a) Pratyush is not guilty of murder because he acted out of sudden and grave provocation. He
immediately tried to stab the shooter but when he failed, he killed the shooter at his house.
(b) Pratyush is guilty of murder because he did not act out of grave and sudden provocation. In
the meanwhile he concerted with his friends and then killed the shooter.
(c) Pratyush is guilty of murder because he had not only killed the shooter but also his wife.
(d) Pratyush is not guilty of murder because he was enraged by the murder of his father and filled
with vengeance while killing the shooter.

85. Legal Principles: Assault is committed when a person makes any gesture, or any preparation in
a convincing way, intending or knowing it to be likely that such gesture or preparation will cause
any person present apprehension of bodily harm.
Factual situation: Suresh loved Pinki and had proposed her but Pinki had refused his proposal.
Pinki was going to marry Ramesh. Learning about Pinki’s and Ramesh’s marriage, Suresh called
Pinki and told her that if she would not become his wife, he would not let her become anyone
else’s wife and told her that if she marries Ramesh, he would kill both Pinki and Ramesh. Pinki
called the police and had him arrested.
(a) Suresh is liable for assault because he created apprehension of death in the mind of Pinki.
(b) Suresh is liable for assault because he created the fear of her own death as well as Ramesh’s
death in the mind of Pinki.
(c) Suresh is not liable for assault because he had merely threatened her by his words on phone
without any gesture or preparation.
(d) Suresh is not liable for assault because he did not cause apprehension of imminent bodily
harm.

Head Office: 127, Zone II, MP Nagar, Bhopal |+91-9111555433| www.legaledge.in Page 21 of 36
MOCK AILET #03

86. Legal Principles: Nothing is an offence which is done by a person who, at the time of doing it, is,
by reason of intoxication, incapable of knowing the nature of the act, or that he is doing what is
either wrong, or contrary to law.
Factual situation: At a friend’s party, Rohan enters a drinking game where everyone has to drink
a shot of tequila every time Barcelona scores a goal on television. In the match that day, Barcelona
scores 8 goals. After the party, Rohan reverses his car from the parking space when something
dashes his car from behind. He gets out of the car, and sees another car, without realizing that
the driver of the car hadn’t moved the car at all till now. He enters into a fit of rage, picks up a brick
lying by and hits the driver while coming out of his car, who dies. The police charges him with
murder; Rohan takes the defence that he was intoxicated. Will Rohan’s defence succeed?

(a) No, since he chose to have alcohol on his own and nobody forced him to have excessive
alcohol.
(b) No, since it is unacceptable that young people get drunk and kill other people.
(c) Yes, since he had had excessive alcohol and did not understand what was happening around
him.
(d) Yes, since the reason for his excessive intoxication was Barcelona’s performance.

87. Legal Principles: No person shall be deprived ofhis property save by authority of legislation.
Factual situation: The District Magistrate of Dastar district issued an order transferring all
privately owned property within two kilometer range of the Kashti river in the district in the name
of the state government for the purpose of converting the said area into an environmental green
belt. The owners of the land filed a petition for quashing of the order under the principle. Will the
petition succeed?

(a) Yes, because the Magistrate should have taken permit from Environment Ministry before
passing such an order.
(b) Yes, because the order by the Magistrate does not have an authority of any legislation.
(c) No, because the Kashti river and surrounding areas have been victims of increasing pollution
over the last few years and unilateral administrative action is required to curb the same.
(d) No, because the Magistrate notified all the affected parties before passing the order.

88. Legal Principles:


I. "Coercion" is the committing, or threatening to commit, any act forbidden by the Indian Penal
Code, or the unlawful detaining, or threatening to detain, any property, to the prejudice of any
person whatever, with the intention of causing any person to enter into an agreement.
Explanation.- It is immaterial whether the Indian Penal Code is or is not in force in the place
where the coercion is employed
II. Section 309 of Indian Penal code provides that whoever attempts to commit suicide and does
any act towards the commission of such offence, shall be punished with simple imprisonment
for a term which may extend to one year 1 or with fine, or with both.
Factual situation: Vidya who lives in Srinagar with her mother Kanta, threatens Kanta that if she
does not sell her house to Vidya for INR 50,000 or less, Vidya will commit suicide. Does Vidya's
act amount to coercion?
(a) No, since Vidya is threatening to commit suicide; the Indian Penal Code on the other hand
provides that only attempt to commit suicide is an offence and therefore actually committing
suicide is not an offence under the Indian Penal Code.
(b) No, since they live in Jammu and Kashmir where the Indian Penal Code is not applicable.
(c) Yes, since Kanta will have to enter the agreement without free consent.
(d) Yes, since there was a threat of attempt to commit suicide
Head Office: 127, Zone II, MP Nagar, Bhopal |+91-9111555433| www.legaledge.in Page 22 of 36
MOCK AILET #03

89. Legal Principles:


I. Whoever counterfeits or knowingly performs any part of the process of counterfeiting coin,
shall be punished with imprisonment of either description for a term which may extend to seven
years, and shall also be liable to fine.
Explanation: A person commits this offence who intending to practise deception, or knowing
it to be likely that deception will thereby be practised, causes a genuine coin to appear like a
different coin.
II. Counterfeiting means causing one thing to resemble another.
Factual situation: Manoj and Raghu have been great friends since childhood and most notorious
kids of their locality. As they grew up, their acts became increasingly callous and anti-social. They
extorted money from passers-by at night, harassed girls and even commit burglary to support their
drinking and gambling habits. One day Raghu came up to Manoj and told him that he had heard
that there was a lot of money in counterfeiting business, specially ever since INR 10 coin was
launched. He could procure raw metal from scrap market at low prices and they could start
manufacturing counterfeit coins. Manoj was excited and told Raghu that he could devise a cast
for minting the coins. Manoj started working on the cast. However, having not had any experience
in such highly sophisticated craft, his cast was poorly made. When Raghu brought the metal and
the two of them tried to mint the INR 10 coin, the resultant product was a very vague imitation of
the actual coin, a travesty. However, having spent much money and effort in the process, they
decided to try and pass off the coins in the market. Their effort was in vain and they were caught
at the very first attempt and charged with counterfeiting the coin. Are they liable?

(a) No, since they were not successful in passing off the coins at all.
(b) No, since in spite of their intent, the products were too different from original coins to constitute
counterfeit.
(c) Yes, since they had a criminal mindset and were a liability on the society.
(d) Yes, since their intention was to pass off the coins as original.

90. Legal Principles:


I. The official language of the Union shall be Hindi in Devanagari script.
II. It shall be the duty of the Union to promote the spread of the Hindi language, to develop it so
that it may serve as a medium of expression for all the elements of the composite culture of
India and to secure its enrichment by assimilating without interfering with its genius, the forms,
style and expressions used in Hindustani and in the other languages of India specified in the
Eighth Schedule, and by drawing, wherever necessary or desirable, for its vocabulary, primarily
on Sanskrit and secondarily on other languages.
III. Notwithstanding anything in the foregoing principles All proceedings in the Supreme Court and
in every High Court, the authoritative texts of all Acts passed by Parliament or the Legislature
of a State and of all Ordinances promulgated by the President or the Governor of a State shall
be in the English language.
Factual situation: Pranav Mahanto, a dedicated academician and propagator of Hindi language,
files a petition alleging that it is a clear principle that Hindi is the official language of the Indian
Union and the Union has been shouldered with the duty to promote it. In this light, the Union is in
breach of its duty by providing that proceedings in Supreme Court and High Courts and Acts
passed by parliament and state legislatures will be in English language. Is there a breach of duty
on part of the Union?

Head Office: 127, Zone II, MP Nagar, Bhopal |+91-9111555433| www.legaledge.in Page 23 of 36
MOCK AILET #03

(a) Union is partly in breach since it is only responsible for matters of the Parliament and Supreme
Court at the central level.
(b) Union is partly in breach since it is only responsible for matters of the Parliament at the central
level.
(c) Union is in breach since the language of proceedings of all the important wings of the
government is at odds with the official language and the duty to promote it.
(d) Union is not in breach since principle governing the language of proceedings as English has
been laid down to operate irrespective of the official language and the duty to promote it.

91. Legal Principle: A copyright may subsist only on the expression of an idea and not the idea itself.
Factual situation: Manu had a path breaking idea for the story of a novel. However, he was not
getting good ideas for a few important plot twists. He approached his friend Tony. After hearing
the idea in some detail including basic outline, character descriptions and the main message, Tony
was not only impressed, but also started thinking about writing his own book based on what he
had just heard. He had even thought about a few more plot twists that were likely to make the
story immensely more enjoyable. Therefore, while he praised Manu on his idea, he presented him
with his inability to contribute and said that he could not think of anything. However, as soon as
Manu was gone, Tony sat down and started writing. He completed and published a novel, which
became instantaneous hit, bringing enormous amount of money in royalties. Manu read Tony's
novel and became enraged on the backstabbing that Tony had committed. He sued him for
copyright infringement. Will his suit succeed?

(a) No, since Tony added his own ideas to what he had heard from Manu.
(b) No, since Tony copied only an idea and not an expression.
(c) Yes, since Manu's talk with Tony constituted verbal expression which Tony copied.
(d) Yes, since Tony had dishonest intention, lied to Manu and stole his potentially lucrative idea.

92. Legal Principle: When two or more persons, by fighting in a public place, disturb the public peace,
they are said to commit an affray.
Factual situation: William and Christina, husband and wife, were having a severe fight in the
terrace of their house. The terrace was only on the first floor and was clearly visible from the street.
The voice from their shouting reached not only the street but most houses in the vicinity.
Eventually, their fight turned violent and Christina started throwing stuff at William who ducked
and a piece of a metal lamp followed by kitchen utensils started falling on the street below.
Although nobody got hurt, it was because everybody on the street was aware of and was watching
the fight. The children in the nearby houses were not able to study and the old and the sick were
not able to sleep because of the fight. Are William and Christina committing affray?

(a) No, since terrace of their house is not a public place.


(b) No, since they are husband and wife and their fight is a domestic affair.
(c) Yes, since the peace-disturbing effects of their fight can be felt in the street as well, which is a
public place.
(d) Yes, since nuisance is being caused in the entire neighbourhood.
93. Legal Principle: There shall be equality of opportunity for all citizens in matters relating to
employment or appointment to any office under the State.
Factual situation: Rashmi applied to Exigency Services Pvt. Ltd. for the post of Financial
Manager. Rashmi had recently completed his MBA in Finance from an Indian Institute of
Management. She had come from a financially weak background and had taken an educational
loan for her studies. However, through hard work, she had managed to earn a prestigious degree
along with learning marketable skillsets. The job at Exigency was her dream job, which gave her
Head Office: 127, Zone II, MP Nagar, Bhopal |+91-9111555433| www.legaledge.in Page 24 of 36
MOCK AILET #03

the right career path and money and one where Rashmi's profile matched perfectly with what had
been advertised. However, while Rashmi gave an excellent interview, the interview panel at
Exigency enquired whether she would be able to work late hours. Rashmi replied that since the
city was not safe for women, she would be able to work late only if the company could make travel
arrangements for her while going back. The company realized that making such arrangements
almost every single day would be a financial burden on the company. There was another male
candidate, Romit, who had slightly less remarkable profile, but still managed to fit the job
requirement. He agreed to staying back late in the office and he had no troubles making his own
arrangements on the way back to home from the office. Exigency gave the job offer to Romit.
Rashmi learnt about the appointment and realized why she must have been denied the job. Deeply
offended, she filed a petition against Exigency that she had not been afforded equality of
opportunity. Will Rashmi's suit succeed?
(a) No, since the company is justified in taking cost factor into account while making appointments.
(b) No, since Exigency is a private company.
(c) Yes, since the decision regarding appointment stemmed from a gender-based distinction.
(d) Yes, since Rashmi can prove that she had better credentials than Romit.
Legal Principles: (Q.94 – Q.96)
I. If the President is satisfied on the basis of a report received by the Governor of a State or
otherwise that a situation has arisen in which the government of the State cannot be carried
on in accordance with the provisions of the Constitution, the same may proclaim emergency
and assume all legislative and executive powers of the State Government.
II. The President must have some material to entertain the belief that State Government cannot
function in accordance with the Constitution. The adequacy or accuracy of the material is
immaterial.
Factual situation: President Y receives certain reports from his office concerning State B,
regarding certain violence among ethnic groups there. He is satisfied that the government of the
State cannot be carried out in accordance with the provisions of the Constitution. Without waiting
for a report from the governor, he proclaims emergency and assumes all powers. There is a similar
composition of ethnic groups in State C. Pre-empting similar strains in C, Y declares emergency
there as well, in the absence of any reports. However, the reports given to the President about
State B turn out to be inaccurate and based on manipulated data.
94. Is the proclamation of emergency in B justified?
(a) Yes, as Y is satisfied about the need for such a proclamation and has the requisite material to
make it.
(b) Yes, because the President is acting in the best interests of the country and is trying to prevent
the ethnic unrest in B.
(c) No, because the material on which the proclamation is based is inaccurate and this makes the
proclamation invalid.
(d) No, because the President should have waited for the governor’s report.
95. Is the proclamation of emergency in C justified?
(a) Yes, as it was made using an analogy drawn from the situation in B, and separate materials
were not required.
(b) No, because there is no material to support such a proclamation and this has been done on
flimsy grounds.
(c) Yes, because the President is trying to act in a proactive manner and has discretionary powers
in such situations.
(d) No, because there is no ostensible unrest in C.

Head Office: 127, Zone II, MP Nagar, Bhopal |+91-9111555433| www.legaledge.in Page 25 of 36
MOCK AILET #03

96. Which of the following High Court will cease the publication of manually printed cause list from
January 1 onwards?
(a) Madras High Court (b) Calcutta High Court
(c) Punjab & Haryana High Court (d) Delhi High Court
97. Supreme Court of which of the following countries has upheld that – “Automatic loss of citizenship
rule is unconstitutional”?
(a) Myanmar (b) Sri Lanka (c) Liberia (d) Ethiopia
98. Which of the following courts issued a notice stating – “Recognize trees as living entities” against
a plea felling of trees in Uttar Pradesh?
(a) Allahabad High Court (b) Delhi High Court
(c) Bombay High Court (d) Supreme Court
99. Which of the following Member of Parliament filed a petition against the Unique Identification
Authority of India (UIDAI) on the tender inviting bids for the ‘social media monitoring agency’?
(a) Mahua Moitra (b) Abdul Khaleque
(c) Pradyut Bordoloi (d) Rajkumar Ranjan Singh
100. Which of the following judges has recused himself/herself from the review petition case of
Nirbhaya gang rape – murder convict, Akshay Kumar Singh?
(a) Justice R. Banumathi (b) Justice Ashok Bhushan
(c) Justice S A Bobde (d) None of the Above
101. Which of the statement/s is/are correct in respect of The Maintenance and Welfare of Parents and
Senior Citizen (Amendment) Bill, 2019?
(a) Criminalising abusing of parents or senior citizens
(b) Expanding the scope of ‘children’ under the act so as to include daughter In law and son in
law.
(c) Both A & B
(d) None of the above
102. Who amongst the following has been elected as the President of Supreme Court Bar
Asssociation?
(a) Indira Jaising (b) Gautam Bhatia
(c) Dushyant Dave (d) Prashant Bhushan
103. Who amongst the following has been appointed as Executive Chairman of National Legal Services
Authority?
(a) Justice S.A Bobde (b) Justice Arun Mishra
(c) Justice DY Chandrachud (d) Justice NV Ramana
104. Which of the following High Courts has become the first in country to provide online certified copies
to litigants?
(a) Telangana High Court (b) Madras High Court
(c) Rajasthan High Court (d) None of the above.
105. Which of the following Member of Parliament is to move the private member’s bill in the Lok Sabha
to nationalise NLUs in India?
(a) Meenakshi Lekhi (b) Mahua Moitra
(c) Abdul Khaleque (d) None of the above.

Head Office: 127, Zone II, MP Nagar, Bhopal |+91-9111555433| www.legaledge.in Page 26 of 36
MOCK AILET #03

SECTION - D: LOGICAL REASONING

Direction (Q.106 - Q.115): Each question describes a situation and is followed by four possible
responses. Indicate the response you find most appropriate. Choose only one response for each
item. The responses will be evaluated based on the level of appropriateness for the given situation.

106. A person lives in a far off village which is almost two hours by bus. The villager's neighbour is a
very powerful landlord who is trying to occupy the poor villager's land by force. You are the District
Magistrate and busy in a meeting called by a local Minister. The villager has come all the way, by
bus and on foot, to see you and give an application seeking protection from the powerful landlord.
The villager keeps on waiting outside the meeting hall for an hour. You come out of the meeting
and are rushing to another meeting. The villager follows you to submit his application. What would
you do?
(a) Tell him to wait for another two hours till you come back from your next meeting.
(b) Tell him that the matter is actually to be dealt by a junior officer and that he should give the
application to him.
(c) Call one of your senior subordinate officers and ask him to solve the villager's problem.
(d) Quickly take the application from him, ask him a few relevant questions regarding his problem
and then proceed to the meeting.

107. There is a shortage of sugar in your District where you are the District Magistrate. The Government
has ordered that only a maximum amount of 30 kg sugar is to be released for wedding
celebrations. A son of your close friend is getting married and your friend requests you to release
at least 50 kg sugar for his son's wedding. He expresses annoyance when you tell him about the
Government's restrictions on this matter. He feels that since you are the District Magistrate you
can release any amount. You do not want to spoil your friendship with him. In such circumstances,
how would you deal with the situation?
(a) Release the extra amount of sugar which your friend has requested for.
(b) Refuse your friend the extra amount and strictly follow the rules.
(c) Show your friend the copy of the Government instructions and then persuade him to accept
the lower amount as prescribed in the rules.
(d) Advise him to directly apply to the allotting authority and inform him that you do not interfere in
this matter.

108. You are in-charge of implementing the Family Planning Programme in an area where there is a
strong opposition to the present policy. You want to convince the residents of the need for keeping
small families. What would be the best way of communicating this message?
(a) By logically explaining to the residents the need for family planning to improve the health and
living standards.
(b) By encouraging late marriages and proper spacing of children.
(c) By offering incentives for adopting family planning devices.
(d) By asking people who have been sterilized or are using contraceptives to directly talk to the
residents.
Use for Rough Work

Head Office: 127, Zone II, MP Nagar, Bhopal |+91-9111555433| www.legaledge.in Page 27 of 36
MOCK AILET #03

109. You are a teacher in a University and are setting a question paper on a particular subject. One of
your colleagues, whose son is preparing for the examination on that subject, comes to you and
informs you that it is his son's last chance to pass that examination and whether you could help
him by indicating what questions are going to be in the examination. In the past, your colleague
had helped you in another matter. Your colleague informs you that his son will suffer from
depression if he fails in this examination. In such circumstances, what would you do?
(a) In view of the help he had given you, extend your help to him.
(b) Regret that you cannot be of any help to him.
(c) Explain to your colleague that this would be violating the trust of the University authorities and
you are not in a position to help him.
(d) Report the conduct of your colleague to the higher authorities

110. It has been reported in recent years that a very large number of seats in engineering colleges in
the country remain vacant at the end of the admission session.
Which of the following may be the probable cause of the above effect?
(a) There has been a considerable decrease in hiring of engineering graduates due to economic
slowdown in the recent years.
(b) Students have always preferred to complete graduation in three years, time instead of four
years for engineering.
(c) The govt. has recently decided to provided post qualification professional training to all
engineering graduates at its own cost
(d) There has always been a very poor success rate among the engineering students.

111. A few travellers were severely beaten up by villagers recently in a remote rural part of the state as
the villagers found the movement of the travellers suspicious. The district authority has sent a
police team to nab the culprits. Which of the following inferences can be drawn from the above
statement? (An inference is something which is not directly stated but can be inferred from the
given facts.)
(a) The villages dislike the presence of strangers in their vicinity.
(b) Villagers are generally suspicious in nature.
(c) Travellers prefer to visit countryside.
(d) The govt. generally provides protection to travellers across the country.

112. There is an alarming trend of skewed sex ratio against women in India. During the past decade
and the situation may go out of hand if adequate steps are not taken to stop female foeticide.
Which of the following can be an effective step to reverse the trend?
(a) The govt. should immediately completely ban the use of scanners /sonography on expecting
mothers at all health centres.
(B) The govt. should announce a substantial incentive scheme for couples who have at least one
girl child
(C) The Goyt should launch a nationwide-campaign to create awareness against female foeticide.
(a) Only (a) (b) Only (A) and (B)
(c) Only (B) and (C) (d) All (A), (B) and (C)
Use for Rough Work

Head Office: 127, Zone II, MP Nagar, Bhopal |+91-9111555433| www.legaledge.in Page 28 of 36
MOCK AILET #03

113. A very large number of technically qualified young Indians are coming out of colleges every year
though there are not enough opportunities for them to get gainful employment.
Which of the following contradicts the views expressed in the above statement?
(a) Technically qualified persons are far superior to those with standard degrees Eke
BA/BSc/B.Com. etc.
(b) The govt. has not done effective planning for engaging technically qualified personnel while
authorizing the setting up of technical colleges.
(c) A huge gap exists between the level of competence of technically qualified graduates and
requirements of the industry:
(d) None of these

114. The govt. has appealed to all citizens to use potable water judiciously as there is an acute shortage
in supply, excessive use may lead to huge scarcity in coming months.
Which of the following assumptions is implicit in the above statement? (An assumption is
something supposed or taken for granted.)
(a) People may ignore the appeal and continue using water as per their consideration.
(b) Govt. may be able to tap those who do not respond to the appeal.
(c) Govt. may be able to put in place alternate sources of water in the event of crisis.
(d) A large number of people may positively respond to the Government’s appeal and help tide
over the crisis.

115. There has been a spate of rail accidents in India in the recent months killing large numbers of
passengers and injuring many more. This has raised serious doubts on the railway’s capability of
providing safety to travellers.
Which of the following statements substantiates the views expressed in the above statement?
(a) Indian Railway is known for providing best comfort to its passenger.
(b) People have no option other than travelling by rail over long distances.
(c) The railway tracks at many places have been found to be stressed due to wear and tear in the
recent times.
(d) Local residents are always the first to provide a helping hand to the passengers in the event
of such disasters.

Directions (Q.116 – Q.120): Read the following information carefully and answer the questions
given below.

A company launches eight products — Q, R, S, T, V, W, Y and Z in one of the four metros of India.
The products were launched one after the other over a period of six months in 2019. The order in
which the products were launched is consistent with the following conditions:
V is launched before both Y and Q.
Q gets launched after Z.
T gets launched before V but after R
S gets launched after V.
R gets launched before W.

116. Which one of the following could be true?


(a) Y is the second product to be launched
(b) S is the fifth product to be launched
(c) Q is the fourth product to be launched
(d) R is the third product to be launched

Head Office: 127, Zone II, MP Nagar, Bhopal |+91-9111555433| www.legaledge.in Page 29 of 36
MOCK AILET #03

117. If Z is the seventh product to be launched, then which one of the following could be true?
(a) T is the second product to be launched
(b) T is the fourth product to be launched
(c) W is the fifth product to be launched
(d) V Is the sixth product to be launched

118. If Q is the fifth product to be launched, then each of the fol¬lowing could be true except
(a) Z is the first product to be launched (b) T is the second product to be launched
(c) V is the third product to be launched (d) W is the fourth product to be launched

119. If R is the second product to be launched, which one of the fol¬lowing must be true?
(a) S gets launched some time before T
(b) T gets launched some time before W
(c) W gets launched some time before V
(d) Z gets launched some time before W

120. If V gets launched before Z does, then which one of the following could be true?
(a) R is the second product to be launched
(b) T is the fourth product to be launched
(c) Q is the fourth product to be launched
(d) Z is the sixth product to be launched

Direction (Q.121 and Q.122): Each of the questions below consists of a question and two
statements numbered I and II given below it. You have to decide whether the data provided in the
statements are sufficient to answer the question.

121. Six persons namely A, B, C, D, E and F teach one subject each, one after another viz., Reasoning,
Quant, Banking, Computer, General awareness and English but not necessarily in the same order.
Quant is not taught before Banking nor it is the last subject taught. Which subject is taught just
after English?
I. The first subject to teach is English which is not be taught by either C or E. A, C and E are not
available to teach the last subject.
II. Reasoning is taught by B just after Banking and just before Computer which is taught by D.
(a) If both the statements I and II taken together are not sufficient to answer the questions.
(b) If statement either I or II is sufficient to answer the question.
(c) If both the statements I and II taken together are sufficient to answer the questions.
(d) If statements I is sufficient to answer the question, but statement II by itself is not sufficient to
answer the question.
Use for Rough Work

Head Office: 127, Zone II, MP Nagar, Bhopal |+91-9111555433| www.legaledge.in Page 30 of 36
MOCK AILET #03

122. Eight friends Sunil, Ravi, Jayant, Kapil, Naren, Shivam, Paresh and Lakshya are seated in a
straight line facing north, but not necessarily in the same order.
Ravi sits second to right of Shivam. Shivam sits at one of the extreme ends of the line, then what
is the position of Kapil?
I. Paresh is not an immediate neighbor of Naren and Jayant sit third to the left of Kapil.
II. Only two persons sit between Kapil and Sunil.
(a) If both the statements I and II taken together are not sufficient to answer the questions.
(b) If statement II by itself is sufficient to answer the question, but statement I alone is not sufficient
to answer the question.
(c) If statement either I or II is sufficient to answer the question.
(d) If statements I is sufficient to answer the question, but statement II by itself is not sufficient to
answer the question.
123. Study the following information carefully and answer the questions which follow
A person X given an exam for the post of Branch manager in a Bank. Candidates must possess
the following criteria. The candidate must—
I. Not less than 35 years and not exceed 40 years as on 01.11.2015.
II. Have a Bachelor's Degree in any discipline with 65 per cent marks.
III. Have a MBA in Banking and finance.
IV. Have experience of at least 4 years in a reputed bank as on 01.12.2016.
V. Relaxation in age of 3 year if candidate is a female.
Study the following information and give the correct sequence from which we can find the exact
answer with minimum number of conditions?
(A) As on 01.11.2016, X’s age is 38 year old.
(B) X has a degree in Engineering and Technology with 72 per cent marks.
(C) Its definite information that X is a Female.
(D) X have Post graduation in MBA with Banking and finance stream.
(E) X is currently working in Axis bank from 15.07.2012.
(a) BECAD (b) ABCDE (c) ABDEC (d) ABECD
Direction (Q.124 and Q.125): In every question two rows are given and to find out the resultant
of a particular row you need to follow the following conditions:
I. If an even number is followed by an odd (prime) number then the resultant will be the addition
of both the numbers.
II. If an odd number is followed by a perfect square then the resultant will be the subtraction of
the square number from the odd number.
III. If an odd number is followed by another odd number then the resultant will be the addition of
both the numbers.
IV. If an even number is followed by an odd (non-prime) number then the resultant will be the
subtraction of the odd number from the even number.
V. If an odd number is followed by an even number then the resultant comes by multiplying the
numbers.
124. 4 5 2
13 9 3
Find the sum of two rows
(a) 18 (b) 25 (c) 11 (d) 14
Use for Rough Work

Head Office: 127, Zone II, MP Nagar, Bhopal |+91-9111555433| www.legaledge.in Page 31 of 36
MOCK AILET #03

125. Find the resultant of second row if X is the resultant of first row.
11 9 7
17 X 3
(a) 11 (b) 13 (c) 10 (d) 9
Directions (Q.126 - Q.128): Study the following information carefully and answer the questions
given below it:
Five family members P, Q, R, S and T are sitting in a row facing north. P is sitting third to the right
of his daughter. Q is sitting next to R, whose husband is the brother of Q. P is sitting next to T.S
is not sitting with P’s son. S is sitting on the left end and the mother of S is R, who is the wife of P.
Q is sitting on the right of R and S.Q and P are sitting together.
126. What is the position of son of R with respect to the father of S in the given seating arrangement?
(a) Immediate left (b) immediate right
(c) third to the right (d) Second to the left

127. How is the daughter of P is related to Q among the family members?


(a) Niece (b) Sister-in-law (c) Wife (d) Sister

128. If the position of Q and T are interchanged, then who among the following does sit immediate right
of S’s mother?
(a) Brother of P (b) P (c) Son of P (d) R

129. Find out missing number:


7 9 8 6 10 9 5 11 10 ? 12
(a) 4 (b) 11 (c) 56 (d) 5

130. Identify the wrong number that does not follow the given pattern.
0, 126, 728, 2178
(a) 728 (b) 2178 (c) 126 (d) 0

131. There are four routes to travel from city A to city B and six routes from city B to city C. How many
routes are possible to travel from the city A to city C?
(a) 24 (b) 12 (c) 10 (d) 8

132. A contract of construction job specifies a penalty for delay in completion of the work beyond a
certain date is as follows:
Rs. 200 for the first day,
Rs. 250 for the second day,
Rs. 300 for the third day etc.,
The penalty for each succeeding day being Rs 50 more than that of the preceding day.
How much penalty should the contractor pay if he delays the work by 10 days?
(a) Rs. 4950 (b) Rs. 4250 (c) Rs. 3600 (d) Rs. 650

Use for Rough Work

Head Office: 127, Zone II, MP Nagar, Bhopal |+91-9111555433| www.legaledge.in Page 32 of 36
MOCK AILET #03

133. Consider the following statements followed by two conclusions :


Statements:
Some men are great.
Some men are wise.
Conclusion I: Men are either great or wise.
Conclusion II: Some men are neither great nor wise.
Which one of the following is correct?
(a) Only conclusion I is valid (b) Only conclusion II is valid
(c) Both the conclusions are valid (d) Neither of the conclusions is valid

134. Consider the following statements :


1. Some claim to have seen UFOs (Unidentified Flying Objects).
2. Life on other heavenly bodies is considered to be a possibility.
3. Voyage to space is now an established fact.
From the above statements, it may be concluded that
(a) UFOs are heavenly bodies
(b) UFOs are sent from other heavenly bodies
(c) Some living species in other heavenly bodies are more intelligent than man
(d) Nothing definite can be said about the UFOs

135. Pointing to a girl, Subodh said "She is the daughter of my mother's only brother". How is Subodh’s
father related to the girl?
(a) Sister (b) Maternal uncle (c) Brother (d) Data inadequate

136. A family has 4 females and 2 males. Further the following information is known:
B is the wife of A and has a son C.
F is the sister of C.
D is the wife of C and has a daughter E.
The family has three generations with A being in the oldest generation.
How is E related to B?
(a) Daughter (b) Daughter-in-law (c) Sister (d) Grand-daughter

137. There are five bags – bag 1, bag 2, bag 3, bag 4 and bag 5. The weight of bag 1 is less than that
of bag 4 but more than that of bag 5. The weight of bag 2 is less than that of bag 3 as well as bag
1. The weight of bag 3 is less than that of bag 5. Which of the following statements is false?
(a) Bag 2 has the least weight
(b) The weight of bag 1 is more than the weight of three bags
(c) The weight of bag 3 is more than that of bag 1
(d) The total weight of bag 1 and bag 5 is more than the total weight of bag 3 and bag 2

Use for Rough Work

Head Office: 127, Zone II, MP Nagar, Bhopal |+91-9111555433| www.legaledge.in Page 33 of 36
MOCK AILET #03

138. Consider the following argument:


Some roads are paths. No path is a street. All streets are highways.

Which one of the following is not a valid conclusion regarding the above argument?
(a) Some highways are streets. (b) Some paths are roads.
(c) Some streets are paths. (d) Some streets are highways.

139. P, Q, R, S and T have different heights. P is neither the tallest nor the shortest. R is either the
tallest or the shortest. Q is ranked among the top two while T is ranked among bottom two in terms
of heights .What is the rank of S if P is not ranked 3?
(a) 5 (b) 2 (c) 3 (d) Cannot be determined

140. Four brothers Raj, Viraj, Dhiraj and Suraj are standing in a row facing north. The eldest brother is
standing second from left. Viraj is youngest and stands at one of the ends. Dhiraj stands between
the eldest brother and Raj. Who stands at the other end of the row?
(a) Dhiraj (b) Suraj (c) Raj (d) Cannot be determined

Use for Rough Work

Head Office: 127, Zone II, MP Nagar, Bhopal |+91-9111555433| www.legaledge.in Page 34 of 36
MOCK AILET #03

SECTION - E : MATHEMATICS

141. Rahul sold an article at Rs.96 in such a way that the profit % and the C.P. are numerically the
same. Then find the cost price.
(a) 60 (b) 45 (c) 55 (d) 48
1
142. The average speed of a train is 4 3 times the average speed of a tractor. The tractor covers 270
km in 15 hours. How much distance will the train cover in 12 hours?
(a) 654 km (b) 896 km (c) 564 km (d) 936 km

143. There are two pumps to fill a tank with water. First pump can fill the empty tank in 8 hours while
the second in 10 hours. If both the pumps are opened at the same time and kept open for 4 hours,
the part of tank that will be filled up is
9 1 2 1
(a) 10 (b) 10 (c) 5 (d) 5

144. The price of rice decreased by 20% and then increased by 50%. By what percent the consumption
need to be increased/decreased so that the expenditure on rice remains same?
(a) 16.67 (b) 20.24 (c) 15.55 (d) 60.12

145. Ratio of present age of Priyam and Deepak is 3: 2, respectively. Before 9 years, ratio of age of
Deepak and Sandeep was 3: 5, respectively and Priyam is 3 years older than Sandeep. What will
be the ratio of age of Deepak and Sandeep after 12 years?
(a) 6: 7 (b) 7: 8 (c) 4: 5 (d) 5: 6

Use for Rough Work

Head Office: 127, Zone II, MP Nagar, Bhopal |+91-9111555433| www.legaledge.in Page 35 of 36
MOCK AILET #03

146. If the circumference of a circle is equal to the perimeter of a rectangle whose breadth is half of its
length, then find the ratio of radius to length.
(a) 3/2𝜋 (b) 3/𝜋 (c) 3/4𝜋 (d) 3/6𝜋
1
147. X and Y borrowed Rs. 2,000 and Rs. 3,000 respectively at the same rate of interest for 2 2 years.
If Y paid Rs.125 more interest than X, find the rate of interest.
(a) 5% (b) 2% (c) 4% (d) 9%

148. A sum of money is lent at a certain rate of compound interest. If, instead the same amount was
lent at the same rate under simple interest, the interest for first two years reduces by Rs 120 and
that for the first three year is Rs 366.Find the sum.
(a) 43500 (b) 48500 (c) 48000 (d) 43800

149. If the number 583_436 is completely divisible by 18, then the smallest whole number in the place
of the blank digit will be how much?
(a) 4 (b) 5 (c) 3 (d) 7

150. Two persons 'A' & 'B' invest Rs. 8000 and Rs. 7000 respectively for a period of 12 months then
find the A’s profit, if the total annual profit earned by them is Rs. 6000?
(a) Rs.3200 (b) Rs.3300 (c) Rs.2500 (d) Rs.2300

Use for Rough Work

Head Office: 127, Zone II, MP Nagar, Bhopal |+91-9111555433| www.legaledge.in Page 36 of 36

You might also like